How to use the barrier method for equality (or positive) constraints?












0












$begingroup$


How to use the barrier method for equality (or positive) constraints?



https://optimization.mccormick.northwestern.edu/index.php/Interior-point_method_for_LP#Barrier_Method



My initial reasoning was that since the example there starts with constraints $f_i(x) leq 0$ and then takes



$$phi(x)=-sum_{i=1}^m log(-f_i(x))$$



Then if one'd start with $f_i(x)=0$, then reasonably one doesn't need to flip the sign so could one then simply do



$$phi(x)=-sum_{i=1}^m log(f_i(x))$$



for constraints of the form $f_i(x)=0$ or $f_i(x) geq 0$.



However, without being entirely familiar with the barrier method, is this enough?










share|cite|improve this question









$endgroup$

















    0












    $begingroup$


    How to use the barrier method for equality (or positive) constraints?



    https://optimization.mccormick.northwestern.edu/index.php/Interior-point_method_for_LP#Barrier_Method



    My initial reasoning was that since the example there starts with constraints $f_i(x) leq 0$ and then takes



    $$phi(x)=-sum_{i=1}^m log(-f_i(x))$$



    Then if one'd start with $f_i(x)=0$, then reasonably one doesn't need to flip the sign so could one then simply do



    $$phi(x)=-sum_{i=1}^m log(f_i(x))$$



    for constraints of the form $f_i(x)=0$ or $f_i(x) geq 0$.



    However, without being entirely familiar with the barrier method, is this enough?










    share|cite|improve this question









    $endgroup$















      0












      0








      0





      $begingroup$


      How to use the barrier method for equality (or positive) constraints?



      https://optimization.mccormick.northwestern.edu/index.php/Interior-point_method_for_LP#Barrier_Method



      My initial reasoning was that since the example there starts with constraints $f_i(x) leq 0$ and then takes



      $$phi(x)=-sum_{i=1}^m log(-f_i(x))$$



      Then if one'd start with $f_i(x)=0$, then reasonably one doesn't need to flip the sign so could one then simply do



      $$phi(x)=-sum_{i=1}^m log(f_i(x))$$



      for constraints of the form $f_i(x)=0$ or $f_i(x) geq 0$.



      However, without being entirely familiar with the barrier method, is this enough?










      share|cite|improve this question









      $endgroup$




      How to use the barrier method for equality (or positive) constraints?



      https://optimization.mccormick.northwestern.edu/index.php/Interior-point_method_for_LP#Barrier_Method



      My initial reasoning was that since the example there starts with constraints $f_i(x) leq 0$ and then takes



      $$phi(x)=-sum_{i=1}^m log(-f_i(x))$$



      Then if one'd start with $f_i(x)=0$, then reasonably one doesn't need to flip the sign so could one then simply do



      $$phi(x)=-sum_{i=1}^m log(f_i(x))$$



      for constraints of the form $f_i(x)=0$ or $f_i(x) geq 0$.



      However, without being entirely familiar with the barrier method, is this enough?







      optimization






      share|cite|improve this question













      share|cite|improve this question











      share|cite|improve this question




      share|cite|improve this question










      asked Jan 30 at 21:09









      mavaviljmavavilj

      2,84911138




      2,84911138






















          1 Answer
          1






          active

          oldest

          votes


















          1












          $begingroup$

          A constraint $f(x) geq 0$ is equivalent to $-f(x) leq 0$. Of course $f$ should be concave if you expect globally optimal solutions.



          An equality constraint does not fit in the barrier method, since the feasible set for such a constraint has a nonempty interior.



          The page you link to also has a description of the primal-dual method. That method can cope with equality constraints (in fact, all inequality constraints are first reformulated as equality constraints).






          share|cite|improve this answer









          $endgroup$













          • $begingroup$
            Would I need to convert the equality into inequality then? And how should I do that?
            $endgroup$
            – mavavilj
            Jan 30 at 21:54












          • $begingroup$
            @mavavilj no, if you transform it into two inequalities, the feasible set is still not full dimensional
            $endgroup$
            – LinAlg
            Jan 30 at 21:54










          • $begingroup$
            Good, but this doesn't answer my question. Since I'm not switching $-f(x)$ to $f(x)$, but $g(-f(x))$ to $g(f(x))$.
            $endgroup$
            – mavavilj
            Feb 25 at 16:56












          • $begingroup$
            @mavavilj the barrier term is $-sum_i log(f_i(x))$ for constraints of the type $f_i(x)geq 0$ (this is obvious from the first line in my answer). However, you need to use a different method for equality constraints, because the logarithmic function forces a strict inequality which you cannot have for an equality constraint.
            $endgroup$
            – LinAlg
            Feb 25 at 17:12












          Your Answer





          StackExchange.ifUsing("editor", function () {
          return StackExchange.using("mathjaxEditing", function () {
          StackExchange.MarkdownEditor.creationCallbacks.add(function (editor, postfix) {
          StackExchange.mathjaxEditing.prepareWmdForMathJax(editor, postfix, [["$", "$"], ["\\(","\\)"]]);
          });
          });
          }, "mathjax-editing");

          StackExchange.ready(function() {
          var channelOptions = {
          tags: "".split(" "),
          id: "69"
          };
          initTagRenderer("".split(" "), "".split(" "), channelOptions);

          StackExchange.using("externalEditor", function() {
          // Have to fire editor after snippets, if snippets enabled
          if (StackExchange.settings.snippets.snippetsEnabled) {
          StackExchange.using("snippets", function() {
          createEditor();
          });
          }
          else {
          createEditor();
          }
          });

          function createEditor() {
          StackExchange.prepareEditor({
          heartbeatType: 'answer',
          autoActivateHeartbeat: false,
          convertImagesToLinks: true,
          noModals: true,
          showLowRepImageUploadWarning: true,
          reputationToPostImages: 10,
          bindNavPrevention: true,
          postfix: "",
          imageUploader: {
          brandingHtml: "Powered by u003ca class="icon-imgur-white" href="https://imgur.com/"u003eu003c/au003e",
          contentPolicyHtml: "User contributions licensed under u003ca href="https://creativecommons.org/licenses/by-sa/3.0/"u003ecc by-sa 3.0 with attribution requiredu003c/au003e u003ca href="https://stackoverflow.com/legal/content-policy"u003e(content policy)u003c/au003e",
          allowUrls: true
          },
          noCode: true, onDemand: true,
          discardSelector: ".discard-answer"
          ,immediatelyShowMarkdownHelp:true
          });


          }
          });














          draft saved

          draft discarded


















          StackExchange.ready(
          function () {
          StackExchange.openid.initPostLogin('.new-post-login', 'https%3a%2f%2fmath.stackexchange.com%2fquestions%2f3094115%2fhow-to-use-the-barrier-method-for-equality-or-positive-constraints%23new-answer', 'question_page');
          }
          );

          Post as a guest















          Required, but never shown

























          1 Answer
          1






          active

          oldest

          votes








          1 Answer
          1






          active

          oldest

          votes









          active

          oldest

          votes






          active

          oldest

          votes









          1












          $begingroup$

          A constraint $f(x) geq 0$ is equivalent to $-f(x) leq 0$. Of course $f$ should be concave if you expect globally optimal solutions.



          An equality constraint does not fit in the barrier method, since the feasible set for such a constraint has a nonempty interior.



          The page you link to also has a description of the primal-dual method. That method can cope with equality constraints (in fact, all inequality constraints are first reformulated as equality constraints).






          share|cite|improve this answer









          $endgroup$













          • $begingroup$
            Would I need to convert the equality into inequality then? And how should I do that?
            $endgroup$
            – mavavilj
            Jan 30 at 21:54












          • $begingroup$
            @mavavilj no, if you transform it into two inequalities, the feasible set is still not full dimensional
            $endgroup$
            – LinAlg
            Jan 30 at 21:54










          • $begingroup$
            Good, but this doesn't answer my question. Since I'm not switching $-f(x)$ to $f(x)$, but $g(-f(x))$ to $g(f(x))$.
            $endgroup$
            – mavavilj
            Feb 25 at 16:56












          • $begingroup$
            @mavavilj the barrier term is $-sum_i log(f_i(x))$ for constraints of the type $f_i(x)geq 0$ (this is obvious from the first line in my answer). However, you need to use a different method for equality constraints, because the logarithmic function forces a strict inequality which you cannot have for an equality constraint.
            $endgroup$
            – LinAlg
            Feb 25 at 17:12
















          1












          $begingroup$

          A constraint $f(x) geq 0$ is equivalent to $-f(x) leq 0$. Of course $f$ should be concave if you expect globally optimal solutions.



          An equality constraint does not fit in the barrier method, since the feasible set for such a constraint has a nonempty interior.



          The page you link to also has a description of the primal-dual method. That method can cope with equality constraints (in fact, all inequality constraints are first reformulated as equality constraints).






          share|cite|improve this answer









          $endgroup$













          • $begingroup$
            Would I need to convert the equality into inequality then? And how should I do that?
            $endgroup$
            – mavavilj
            Jan 30 at 21:54












          • $begingroup$
            @mavavilj no, if you transform it into two inequalities, the feasible set is still not full dimensional
            $endgroup$
            – LinAlg
            Jan 30 at 21:54










          • $begingroup$
            Good, but this doesn't answer my question. Since I'm not switching $-f(x)$ to $f(x)$, but $g(-f(x))$ to $g(f(x))$.
            $endgroup$
            – mavavilj
            Feb 25 at 16:56












          • $begingroup$
            @mavavilj the barrier term is $-sum_i log(f_i(x))$ for constraints of the type $f_i(x)geq 0$ (this is obvious from the first line in my answer). However, you need to use a different method for equality constraints, because the logarithmic function forces a strict inequality which you cannot have for an equality constraint.
            $endgroup$
            – LinAlg
            Feb 25 at 17:12














          1












          1








          1





          $begingroup$

          A constraint $f(x) geq 0$ is equivalent to $-f(x) leq 0$. Of course $f$ should be concave if you expect globally optimal solutions.



          An equality constraint does not fit in the barrier method, since the feasible set for such a constraint has a nonempty interior.



          The page you link to also has a description of the primal-dual method. That method can cope with equality constraints (in fact, all inequality constraints are first reformulated as equality constraints).






          share|cite|improve this answer









          $endgroup$



          A constraint $f(x) geq 0$ is equivalent to $-f(x) leq 0$. Of course $f$ should be concave if you expect globally optimal solutions.



          An equality constraint does not fit in the barrier method, since the feasible set for such a constraint has a nonempty interior.



          The page you link to also has a description of the primal-dual method. That method can cope with equality constraints (in fact, all inequality constraints are first reformulated as equality constraints).







          share|cite|improve this answer












          share|cite|improve this answer



          share|cite|improve this answer










          answered Jan 30 at 21:31









          LinAlgLinAlg

          10.1k1521




          10.1k1521












          • $begingroup$
            Would I need to convert the equality into inequality then? And how should I do that?
            $endgroup$
            – mavavilj
            Jan 30 at 21:54












          • $begingroup$
            @mavavilj no, if you transform it into two inequalities, the feasible set is still not full dimensional
            $endgroup$
            – LinAlg
            Jan 30 at 21:54










          • $begingroup$
            Good, but this doesn't answer my question. Since I'm not switching $-f(x)$ to $f(x)$, but $g(-f(x))$ to $g(f(x))$.
            $endgroup$
            – mavavilj
            Feb 25 at 16:56












          • $begingroup$
            @mavavilj the barrier term is $-sum_i log(f_i(x))$ for constraints of the type $f_i(x)geq 0$ (this is obvious from the first line in my answer). However, you need to use a different method for equality constraints, because the logarithmic function forces a strict inequality which you cannot have for an equality constraint.
            $endgroup$
            – LinAlg
            Feb 25 at 17:12


















          • $begingroup$
            Would I need to convert the equality into inequality then? And how should I do that?
            $endgroup$
            – mavavilj
            Jan 30 at 21:54












          • $begingroup$
            @mavavilj no, if you transform it into two inequalities, the feasible set is still not full dimensional
            $endgroup$
            – LinAlg
            Jan 30 at 21:54










          • $begingroup$
            Good, but this doesn't answer my question. Since I'm not switching $-f(x)$ to $f(x)$, but $g(-f(x))$ to $g(f(x))$.
            $endgroup$
            – mavavilj
            Feb 25 at 16:56












          • $begingroup$
            @mavavilj the barrier term is $-sum_i log(f_i(x))$ for constraints of the type $f_i(x)geq 0$ (this is obvious from the first line in my answer). However, you need to use a different method for equality constraints, because the logarithmic function forces a strict inequality which you cannot have for an equality constraint.
            $endgroup$
            – LinAlg
            Feb 25 at 17:12
















          $begingroup$
          Would I need to convert the equality into inequality then? And how should I do that?
          $endgroup$
          – mavavilj
          Jan 30 at 21:54






          $begingroup$
          Would I need to convert the equality into inequality then? And how should I do that?
          $endgroup$
          – mavavilj
          Jan 30 at 21:54














          $begingroup$
          @mavavilj no, if you transform it into two inequalities, the feasible set is still not full dimensional
          $endgroup$
          – LinAlg
          Jan 30 at 21:54




          $begingroup$
          @mavavilj no, if you transform it into two inequalities, the feasible set is still not full dimensional
          $endgroup$
          – LinAlg
          Jan 30 at 21:54












          $begingroup$
          Good, but this doesn't answer my question. Since I'm not switching $-f(x)$ to $f(x)$, but $g(-f(x))$ to $g(f(x))$.
          $endgroup$
          – mavavilj
          Feb 25 at 16:56






          $begingroup$
          Good, but this doesn't answer my question. Since I'm not switching $-f(x)$ to $f(x)$, but $g(-f(x))$ to $g(f(x))$.
          $endgroup$
          – mavavilj
          Feb 25 at 16:56














          $begingroup$
          @mavavilj the barrier term is $-sum_i log(f_i(x))$ for constraints of the type $f_i(x)geq 0$ (this is obvious from the first line in my answer). However, you need to use a different method for equality constraints, because the logarithmic function forces a strict inequality which you cannot have for an equality constraint.
          $endgroup$
          – LinAlg
          Feb 25 at 17:12




          $begingroup$
          @mavavilj the barrier term is $-sum_i log(f_i(x))$ for constraints of the type $f_i(x)geq 0$ (this is obvious from the first line in my answer). However, you need to use a different method for equality constraints, because the logarithmic function forces a strict inequality which you cannot have for an equality constraint.
          $endgroup$
          – LinAlg
          Feb 25 at 17:12


















          draft saved

          draft discarded




















































          Thanks for contributing an answer to Mathematics Stack Exchange!


          • Please be sure to answer the question. Provide details and share your research!

          But avoid



          • Asking for help, clarification, or responding to other answers.

          • Making statements based on opinion; back them up with references or personal experience.


          Use MathJax to format equations. MathJax reference.


          To learn more, see our tips on writing great answers.




          draft saved


          draft discarded














          StackExchange.ready(
          function () {
          StackExchange.openid.initPostLogin('.new-post-login', 'https%3a%2f%2fmath.stackexchange.com%2fquestions%2f3094115%2fhow-to-use-the-barrier-method-for-equality-or-positive-constraints%23new-answer', 'question_page');
          }
          );

          Post as a guest















          Required, but never shown





















































          Required, but never shown














          Required, but never shown












          Required, but never shown







          Required, but never shown

































          Required, but never shown














          Required, but never shown












          Required, but never shown







          Required, but never shown







          Popular posts from this blog

          Can a sorcerer learn a 5th-level spell early by creating spell slots using the Font of Magic feature?

          Does disintegrating a polymorphed enemy still kill it after the 2018 errata?

          A Topological Invariant for $pi_3(U(n))$